Indukcja nierowność

Ze względu na specyfikę metody - osobny dział.
kiler7
Użytkownik
Użytkownik
Posty: 82
Rejestracja: 5 maja 2010, o 16:32
Płeć: Mężczyzna
Lokalizacja: wawa
Podziękował: 10 razy
Pomógł: 1 raz

Indukcja nierowność

Post autor: kiler7 »

Udowodnij korzystając z np indukcji

\(\displaystyle{ 3^n>n^3 ,n \ge 4}\)

Sprawdzilem najpierw dla czterech, potem teza:

\(\displaystyle{ 3^(n+1)>(n+1)^3}\)

Rozpisałem do :

\(\displaystyle{ 3^n*3>n^3+3n^2+3n+1}\)

Prosze o pomoc nie wiem co dalej robic
adambak
Użytkownik
Użytkownik
Posty: 1272
Rejestracja: 8 sty 2011, o 18:18
Płeć: Mężczyzna
Lokalizacja: Warszawa
Podziękował: 295 razy
Pomógł: 115 razy

Indukcja nierowność

Post autor: adambak »

pomnóż nierówność z zadania przez \(\displaystyle{ 3}\).. potem wystarczy dowieść, że \(\displaystyle{ 3n^3>(n+1)^3}\) dla takich \(\displaystyle{ n}\) z zadania..
szw1710

Indukcja nierowność

Post autor: szw1710 »

Można też inaczej: logarytmując mamy równoważnie

\(\displaystyle{ n\ln 3>3\ln n}\), więc

\(\displaystyle{ \frac{\ln 3}{3}>\frac{\ln n}{n}}\).

Niech teraz \(\displaystyle{ f(x)=\frac{\ln{x}}{x}}\), stąd \(\displaystyle{ f'(x)=\frac{1-\ln x}{x^2}<0\iff \ln x>1\iff x>\text{e}}\) , więc w liczbach naturalnych mamy \(\displaystyle{ x\ge 3.}\)

Oznacza to, że \(\displaystyle{ f}\) maleje w \(\displaystyle{ [3,infty)}\), skąd \(\displaystyle{ f(3)>f(n)}\) dla \(\displaystyle{ n\ge 4}\), co daje Twoją nierówność.
kiler7
Użytkownik
Użytkownik
Posty: 82
Rejestracja: 5 maja 2010, o 16:32
Płeć: Mężczyzna
Lokalizacja: wawa
Podziękował: 10 razy
Pomógł: 1 raz

Indukcja nierowność

Post autor: kiler7 »

adambak pisze:pomnóż nierówność z zadania przez \(\displaystyle{ 3}\).. potem wystarczy dowieść, że \(\displaystyle{ 3n^3>(n+1)^3}\) dla takich \(\displaystyle{ n}\) z zadania..

Jak byś mógł bardziej mi to rozpisać choć 2,3 linijki co do rozw szw1710 nie moj poziom jeszcze ;]

Szkoda ze nie da sie dowodu indukcja przejść a tą ja mieliśmy na zajęciach,
szw1710

Indukcja nierowność

Post autor: szw1710 »

Jak najbardziej to zadanie na dowód indukcyjny. Ale ja nie znoszę indukcji tam, gdzie jej nie trzeba. Więc niech się pomęczą młodsi Koledzy, jeśli oczywiście mają na to ochotę

Pochodnych jeszcze nie miałeś (i badania monotoniczności funkcji za pomocą pochodnych)?
kiler7
Użytkownik
Użytkownik
Posty: 82
Rejestracja: 5 maja 2010, o 16:32
Płeć: Mężczyzna
Lokalizacja: wawa
Podziękował: 10 razy
Pomógł: 1 raz

Indukcja nierowność

Post autor: kiler7 »

szw1710 pisze:Jak najbardziej to zadanie na dowód indukcyjny. Ale ja nie znoszę indukcji tam, gdzie jej nie trzeba. Więc niech się pomęczą młodsi Koledzy, jeśli oczywiście mają na to ochotę

Pochodnych jeszcze nie miałeś (i badania monotoniczności funkcji za pomocą pochodnych)?
Dzięki za pomoc, liczę na młodzież.

Według programu nie, chodz sam trochę poznałem
Marcinek665
Użytkownik
Użytkownik
Posty: 1824
Rejestracja: 11 sty 2007, o 20:12
Płeć: Mężczyzna
Lokalizacja: Katowice, Warszawa
Podziękował: 73 razy
Pomógł: 228 razy

Indukcja nierowność

Post autor: Marcinek665 »

Dowodzenie nierówności całkowitoliczbowych za pomocą pochodnej nie jest fajne.

EDIT. Przepraszam, że wyraziłem własne zdanie na temat rozwiązania zadania za pomocą brzydkiej metody.
Ostatnio zmieniony 26 paź 2011, o 22:10 przez Marcinek665, łącznie zmieniany 3 razy.
kamil13151
Użytkownik
Użytkownik
Posty: 5018
Rejestracja: 28 wrz 2009, o 16:53
Płeć: Mężczyzna
Lokalizacja: Warszawa
Podziękował: 459 razy
Pomógł: 912 razy

Indukcja nierowność

Post autor: kamil13151 »

Zakładamy, że nierówność \(\displaystyle{ 3^n>n^3 ,n \ge 4}\) jest prawdziwa, również przy okazji zauważamy, że \(\displaystyle{ 3^{n+1}>3n^3}\) jest prawdziwa. Musimy udowodnić, że \(\displaystyle{ 3^{n+1}>(n+1)^3}\). Wystarczy udowodnić, że: \(\displaystyle{ 3^{n+1}>3n^3>(n+1)^3}\), pierwsza nierówność jest prawdziwa z założenia, także zostaje nam rozwiązać. \(\displaystyle{ 3n^3>(n+1)^3}\), nierówności umiesz chyba już rozwiązać?
adambak
Użytkownik
Użytkownik
Posty: 1272
Rejestracja: 8 sty 2011, o 18:18
Płeć: Mężczyzna
Lokalizacja: Warszawa
Podziękował: 295 razy
Pomógł: 115 razy

Indukcja nierowność

Post autor: adambak »

kiler7, nie chciałbym Cię rozczarować, ale też użyłem do tego pochodnych.. ale fajnie się liczyło..

\(\displaystyle{ 3n^3>(n+1)^3}\)

\(\displaystyle{ 2n^3-3n^2-3n-1>0}\)

\(\displaystyle{ f(n)=2n^3-3n^2-3n-1>0, \ f(4)>0}\)

no to badamy tą funkcję, czy spełnia warunki że dla czwórki wzwyż będzie tylko dodatnia..

\(\displaystyle{ f'(n)=6n^2-6n-3}\)

teraz tylko liczymy deltę i widzimy że nie dość że \(\displaystyle{ f(4)>0}\) to w dodatku pochodna jest dodatnia na przedziale \(\displaystyle{ (\frac{1+\sqrt3}{2};+\infty)}\), a więc funkcja już tylko rośnie co nam wystarcza..
kiler7
Użytkownik
Użytkownik
Posty: 82
Rejestracja: 5 maja 2010, o 16:32
Płeć: Mężczyzna
Lokalizacja: wawa
Podziękował: 10 razy
Pomógł: 1 raz

Indukcja nierowność

Post autor: kiler7 »

kamil13151 pisze:Zakładamy, że nierówność \(\displaystyle{ 3^n>n^3 ,n \ge 4}\) jest prawdziwa, również przy okazji zauważamy, że \(\displaystyle{ 3^{n+1}>3n^3}\) jest prawdziwa. Musimy udowodnić, że \(\displaystyle{ 3^{n+1}>(n+1)^3}\). Wystarczy udowodnić, że: \(\displaystyle{ 3^{n+1}>3n^3>(n+1)^3}\), pierwsza nierówność jest prawdziwa z założenia, także zostaje nam rozwiązać. \(\displaystyle{ 3n^3>(n+1)^3}\), nierówności umiesz chyba już rozwiązać?

Tak defakto to w 1 poscie moim doszedłem do \(\displaystyle{ 3n^3>(n+1)^3}\) i cos tam starałem się rozpisać ale mi nie wychodziło
kamil13151
Użytkownik
Użytkownik
Posty: 5018
Rejestracja: 28 wrz 2009, o 16:53
Płeć: Mężczyzna
Lokalizacja: Warszawa
Podziękował: 459 razy
Pomógł: 912 razy

Indukcja nierowność

Post autor: kamil13151 »

\(\displaystyle{ f(4)>0}\) i \(\displaystyle{ \lim_{n \to \infty } 2n^3-3n^2-3n-1= \infty}\) To chyba wystarcza? .
kiler7
Użytkownik
Użytkownik
Posty: 82
Rejestracja: 5 maja 2010, o 16:32
Płeć: Mężczyzna
Lokalizacja: wawa
Podziękował: 10 razy
Pomógł: 1 raz

Indukcja nierowność

Post autor: kiler7 »

kamil13151 pisze:\(\displaystyle{ f(4)>0}\) i \(\displaystyle{ \lim_{n \to \infty } 2n^3-3n^2-3n-1= \infty}\) To chyba wystarcza? .

Tak ,ale dowód nadal nie indukcyjny tresc moze nie wymaga ale czlowiek dla sportu by chicał to indukcyjnie pokazac
kamil13151
Użytkownik
Użytkownik
Posty: 5018
Rejestracja: 28 wrz 2009, o 16:53
Płeć: Mężczyzna
Lokalizacja: Warszawa
Podziękował: 459 razy
Pomógł: 912 razy

Indukcja nierowność

Post autor: kamil13151 »

To dowód indukcyjny, nierówność \(\displaystyle{ 3n^3>(n+1)^3}\) uzasadniłem dla \(\displaystyle{ n \ge 4}\), ale wcześniej powołałem się na indukcję.
adambak
Użytkownik
Użytkownik
Posty: 1272
Rejestracja: 8 sty 2011, o 18:18
Płeć: Mężczyzna
Lokalizacja: Warszawa
Podziękował: 295 razy
Pomógł: 115 razy

Indukcja nierowność

Post autor: adambak »

kamil13151 pisze:\(\displaystyle{ f(4)>0}\) i \(\displaystyle{ \lim_{n \to \infty } 2n^3-3n^2-3n-1= \infty}\) To chyba wystarcza? .
ja bym się bał mówić czy wystarcza.. funkcja mogłaby jeszcze zakręcić (choć wiem, że wiesz, że nie zakręca )
olaliście chyba mój post, w którym jest dosyć ściśle napisane co nam wystarczy do dowodu

kiler7, to jest indukcja, tylko taka piękna w połączeniu z analizą.. ja się cieszę jak widzę zadanie w którym muszę użyć kilku narzędzi..
kamil13151
Użytkownik
Użytkownik
Posty: 5018
Rejestracja: 28 wrz 2009, o 16:53
Płeć: Mężczyzna
Lokalizacja: Warszawa
Podziękował: 459 razy
Pomógł: 912 razy

Indukcja nierowność

Post autor: kamil13151 »

adambak, no właśnie też się boję, dlatego na koniec wypowiedzi dałem znak zapytania czy wystarcza .
ODPOWIEDZ